If $x_1,x_2,ldots,x_n$ are the roots for $1+x+x^2+ldots+x^n=0$, find the value of...
$begingroup$
Let $x_1,x_2,ldots,x_n$ be the roots for $1+x+x^2+ldots+x^n=0$. Find the value of
$$P(1)=frac{1}{x_1-1}+frac{1}{x_2-1}+ldots+frac{1}{x_n-1}$$
Source: IME entrance exam (Military Institute of Engineering, Brazil), date not provided (possibly from the 1950s)
My attempt:
Developing expression $P(1)$, replacing the 1 by $x$, follows
$$P(x)=frac{(x_2-x)cdots (x_n-x)+ldots+(x_1-x)cdots (x_{n-1}-x)}{(x_1-x)(x_2-x)cdots (x_n-x)}$$
As $x_1,x_2,ldots,x_n$ are the roots, it must be true that
$$Q(x)=(x-x_1)cdots(x-x_n)=1+x+x^2+ldots+x^n$$
and
$$Q(1)=(1-x_1)cdots(1-x_n)=n+1$$
Therefore the denominator of $P(1)$ is
$$(-1)^{n} (n+1).$$ But I could not find a way to simplify the numerator.
Another fact that is probably useful is that
$$1+x^{n+1}=(1-x)(x^n+x^{n-1}+ldots+x+1)$$
with roots that are 1 in addition of the given roots $x_1,x_2,ldots,x_n$ for the original equation, that is
$$x_k=text{cis}(frac{2kpi}{n+1}), k=1,ldots,n.$$
This is as far as I could go...
Hints and full answers are welcomed.
algebra-precalculus complex-numbers contest-math
$endgroup$
add a comment |
$begingroup$
Let $x_1,x_2,ldots,x_n$ be the roots for $1+x+x^2+ldots+x^n=0$. Find the value of
$$P(1)=frac{1}{x_1-1}+frac{1}{x_2-1}+ldots+frac{1}{x_n-1}$$
Source: IME entrance exam (Military Institute of Engineering, Brazil), date not provided (possibly from the 1950s)
My attempt:
Developing expression $P(1)$, replacing the 1 by $x$, follows
$$P(x)=frac{(x_2-x)cdots (x_n-x)+ldots+(x_1-x)cdots (x_{n-1}-x)}{(x_1-x)(x_2-x)cdots (x_n-x)}$$
As $x_1,x_2,ldots,x_n$ are the roots, it must be true that
$$Q(x)=(x-x_1)cdots(x-x_n)=1+x+x^2+ldots+x^n$$
and
$$Q(1)=(1-x_1)cdots(1-x_n)=n+1$$
Therefore the denominator of $P(1)$ is
$$(-1)^{n} (n+1).$$ But I could not find a way to simplify the numerator.
Another fact that is probably useful is that
$$1+x^{n+1}=(1-x)(x^n+x^{n-1}+ldots+x+1)$$
with roots that are 1 in addition of the given roots $x_1,x_2,ldots,x_n$ for the original equation, that is
$$x_k=text{cis}(frac{2kpi}{n+1}), k=1,ldots,n.$$
This is as far as I could go...
Hints and full answers are welcomed.
algebra-precalculus complex-numbers contest-math
$endgroup$
$begingroup$
Related : math.stackexchange.com/questions/2988339/…
$endgroup$
– Arnaud D.
Dec 17 '18 at 12:07
add a comment |
$begingroup$
Let $x_1,x_2,ldots,x_n$ be the roots for $1+x+x^2+ldots+x^n=0$. Find the value of
$$P(1)=frac{1}{x_1-1}+frac{1}{x_2-1}+ldots+frac{1}{x_n-1}$$
Source: IME entrance exam (Military Institute of Engineering, Brazil), date not provided (possibly from the 1950s)
My attempt:
Developing expression $P(1)$, replacing the 1 by $x$, follows
$$P(x)=frac{(x_2-x)cdots (x_n-x)+ldots+(x_1-x)cdots (x_{n-1}-x)}{(x_1-x)(x_2-x)cdots (x_n-x)}$$
As $x_1,x_2,ldots,x_n$ are the roots, it must be true that
$$Q(x)=(x-x_1)cdots(x-x_n)=1+x+x^2+ldots+x^n$$
and
$$Q(1)=(1-x_1)cdots(1-x_n)=n+1$$
Therefore the denominator of $P(1)$ is
$$(-1)^{n} (n+1).$$ But I could not find a way to simplify the numerator.
Another fact that is probably useful is that
$$1+x^{n+1}=(1-x)(x^n+x^{n-1}+ldots+x+1)$$
with roots that are 1 in addition of the given roots $x_1,x_2,ldots,x_n$ for the original equation, that is
$$x_k=text{cis}(frac{2kpi}{n+1}), k=1,ldots,n.$$
This is as far as I could go...
Hints and full answers are welcomed.
algebra-precalculus complex-numbers contest-math
$endgroup$
Let $x_1,x_2,ldots,x_n$ be the roots for $1+x+x^2+ldots+x^n=0$. Find the value of
$$P(1)=frac{1}{x_1-1}+frac{1}{x_2-1}+ldots+frac{1}{x_n-1}$$
Source: IME entrance exam (Military Institute of Engineering, Brazil), date not provided (possibly from the 1950s)
My attempt:
Developing expression $P(1)$, replacing the 1 by $x$, follows
$$P(x)=frac{(x_2-x)cdots (x_n-x)+ldots+(x_1-x)cdots (x_{n-1}-x)}{(x_1-x)(x_2-x)cdots (x_n-x)}$$
As $x_1,x_2,ldots,x_n$ are the roots, it must be true that
$$Q(x)=(x-x_1)cdots(x-x_n)=1+x+x^2+ldots+x^n$$
and
$$Q(1)=(1-x_1)cdots(1-x_n)=n+1$$
Therefore the denominator of $P(1)$ is
$$(-1)^{n} (n+1).$$ But I could not find a way to simplify the numerator.
Another fact that is probably useful is that
$$1+x^{n+1}=(1-x)(x^n+x^{n-1}+ldots+x+1)$$
with roots that are 1 in addition of the given roots $x_1,x_2,ldots,x_n$ for the original equation, that is
$$x_k=text{cis}(frac{2kpi}{n+1}), k=1,ldots,n.$$
This is as far as I could go...
Hints and full answers are welcomed.
algebra-precalculus complex-numbers contest-math
algebra-precalculus complex-numbers contest-math
edited Dec 14 '18 at 9:17
bluemaster
asked Dec 13 '18 at 19:23
bluemasterbluemaster
1,653519
1,653519
$begingroup$
Related : math.stackexchange.com/questions/2988339/…
$endgroup$
– Arnaud D.
Dec 17 '18 at 12:07
add a comment |
$begingroup$
Related : math.stackexchange.com/questions/2988339/…
$endgroup$
– Arnaud D.
Dec 17 '18 at 12:07
$begingroup$
Related : math.stackexchange.com/questions/2988339/…
$endgroup$
– Arnaud D.
Dec 17 '18 at 12:07
$begingroup$
Related : math.stackexchange.com/questions/2988339/…
$endgroup$
– Arnaud D.
Dec 17 '18 at 12:07
add a comment |
5 Answers
5
active
oldest
votes
$begingroup$
$1$ and $x_1,ldots,x_n$ are the roots of $z^{n+1}-1=0$. They are
the $(n+1)$-th roots of unity.
Then $1/(z^{n+1}-1)$
has a partial fraction expansion of the form
$$frac1{z^{n+1}-1}=frac a{z-1}+sum_{k=1}^nfrac{b_k}{x_kz-1}.$$
Multiplying by $z-1$ and setting $z=1$ gives $a=1/(n+1)$.
Multiplying by $x_kz-1$ and setting $z=1/x_k$ gives $b_k=1/(n+1)$.
Therefore
$$sum_{k=1}^nfrac1{x_kz-1}=frac{n+1}{z^{n+1}-1}-frac1{z-1}.$$
Letting $zto1$ gives
$$sum_{k=1}^nfrac1{x_k-1}=lim_{zto1}frac{n-z-z^2-cdots-z^n}{z^{n+1}-1}
=frac{-1-2-cdots-n}{n+1}=-frac n2.$$
ADDED IN EDIT.
Here's a trick proof. Let $S$ denote the sum in question. As the
reciprocals of the $x_k$ are the $x_k$ again (in a different order) then
$$S=sum_{k=1}^nfrac1{x_k^{-1}-1}=sum_{k=1}^nfrac{x_k}{1-x_k}
$$
and
$$2S=sum_{k=1}^nleft(frac1{x_k-1}+frac{x_k}{1-x_k}right)
=sum_{k=1}^n(-1)=-n.$$
$endgroup$
add a comment |
$begingroup$
Applying the same technique as per this answer
$$Q(x)=prodlimits_{k=1}^n(x-x_k) text{ and }
Q'(x)=sumlimits_{i=1}^{n}prodlimits_{k=1,kne i}^{n}left(x-x_kright)$$
then
$$frac{Q'(x)}{Q(x)}=sumlimits_{i=1}^{n}frac{1}{x-x_i}$$
or
$$-frac{Q'(1)}{Q(1)}=sumlimits_{i=1}^{n}frac{1}{x_i-1}$$
But $Q(1)=n+1$ and $Q'(x)=1+2x+3x^3+...+nx^{n-1}Rightarrow Q'(1)=frac{n(n+1)}{2}$ and
$$sumlimits_{i=1}^{n}frac{1}{x_i-1}=-frac{n}{2}$$
$endgroup$
add a comment |
$begingroup$
We have $$x^{n+1}-1=0 (1)$$ with $xne1$
Set $dfrac1{x-1}=yiff x=dfrac{y+1}y$
Replace the value of $x$ in terms of $y$ in $(1)$ to form an $n$ degree equation in $y$ $$left(dfrac{y+1}yright)^{n+1}=1$$
$$impliesbinom{n+1}1 y^n+binom{n+1}2 y^{n-1}+cdots+1=0$$
Now apply Vieta's formula to find $$sum_{r=1}^ndfrac1{x_r-1}=sum_{r=1}^ny_r=-dfrac{binom{n+1}2}{binom{n+1}1}=?$$
$endgroup$
add a comment |
$begingroup$
Simply note that $1,x_1,ldots, x_n$ are roots of $$
x^{n+1} -1 = 0.$$ Thus we have $0,x_1-1,ldots, x_n-1$ are roots of
$$
(x+1)^{n+1} -1 = xsum_{j=0}^{n} binom{n+1}{j+1}x^j = 0.
$$
Hence, $z_i = x_i -1$, $1leq i leq n$ are roots of
$$
sum_{j=0}^{n} binom{n+1}{j+1}z^j = 0,
$$ and by Vieta's formula,
$$
sum_{i=1}^n frac{1}{z_i} = frac{sum_{i=1}^n z_1z_2cdots z_{i-1}widehat{z_i}z_{i+1}cdots z_n}{z_1z_2cdots z_{n-1}z_n} = -frac{binom{n+1}{2}}{binom{n+1}{1}} = -frac{n}{2},
$$ where $widehat{z_i}$ means the variable is omitted in the calculation.
$endgroup$
add a comment |
$begingroup$
First, the denominator is, I think, $(-1)^nQ(1)$.
Here is a hint for the numerator: how can you write $Q’(x)$?
$endgroup$
add a comment |
Your Answer
StackExchange.ifUsing("editor", function () {
return StackExchange.using("mathjaxEditing", function () {
StackExchange.MarkdownEditor.creationCallbacks.add(function (editor, postfix) {
StackExchange.mathjaxEditing.prepareWmdForMathJax(editor, postfix, [["$", "$"], ["\\(","\\)"]]);
});
});
}, "mathjax-editing");
StackExchange.ready(function() {
var channelOptions = {
tags: "".split(" "),
id: "69"
};
initTagRenderer("".split(" "), "".split(" "), channelOptions);
StackExchange.using("externalEditor", function() {
// Have to fire editor after snippets, if snippets enabled
if (StackExchange.settings.snippets.snippetsEnabled) {
StackExchange.using("snippets", function() {
createEditor();
});
}
else {
createEditor();
}
});
function createEditor() {
StackExchange.prepareEditor({
heartbeatType: 'answer',
autoActivateHeartbeat: false,
convertImagesToLinks: true,
noModals: true,
showLowRepImageUploadWarning: true,
reputationToPostImages: 10,
bindNavPrevention: true,
postfix: "",
imageUploader: {
brandingHtml: "Powered by u003ca class="icon-imgur-white" href="https://imgur.com/"u003eu003c/au003e",
contentPolicyHtml: "User contributions licensed under u003ca href="https://creativecommons.org/licenses/by-sa/3.0/"u003ecc by-sa 3.0 with attribution requiredu003c/au003e u003ca href="https://stackoverflow.com/legal/content-policy"u003e(content policy)u003c/au003e",
allowUrls: true
},
noCode: true, onDemand: true,
discardSelector: ".discard-answer"
,immediatelyShowMarkdownHelp:true
});
}
});
Sign up or log in
StackExchange.ready(function () {
StackExchange.helpers.onClickDraftSave('#login-link');
});
Sign up using Google
Sign up using Facebook
Sign up using Email and Password
Post as a guest
Required, but never shown
StackExchange.ready(
function () {
StackExchange.openid.initPostLogin('.new-post-login', 'https%3a%2f%2fmath.stackexchange.com%2fquestions%2f3038472%2fif-x-1-x-2-ldots-x-n-are-the-roots-for-1xx2-ldotsxn-0-find-the-value%23new-answer', 'question_page');
}
);
Post as a guest
Required, but never shown
5 Answers
5
active
oldest
votes
5 Answers
5
active
oldest
votes
active
oldest
votes
active
oldest
votes
$begingroup$
$1$ and $x_1,ldots,x_n$ are the roots of $z^{n+1}-1=0$. They are
the $(n+1)$-th roots of unity.
Then $1/(z^{n+1}-1)$
has a partial fraction expansion of the form
$$frac1{z^{n+1}-1}=frac a{z-1}+sum_{k=1}^nfrac{b_k}{x_kz-1}.$$
Multiplying by $z-1$ and setting $z=1$ gives $a=1/(n+1)$.
Multiplying by $x_kz-1$ and setting $z=1/x_k$ gives $b_k=1/(n+1)$.
Therefore
$$sum_{k=1}^nfrac1{x_kz-1}=frac{n+1}{z^{n+1}-1}-frac1{z-1}.$$
Letting $zto1$ gives
$$sum_{k=1}^nfrac1{x_k-1}=lim_{zto1}frac{n-z-z^2-cdots-z^n}{z^{n+1}-1}
=frac{-1-2-cdots-n}{n+1}=-frac n2.$$
ADDED IN EDIT.
Here's a trick proof. Let $S$ denote the sum in question. As the
reciprocals of the $x_k$ are the $x_k$ again (in a different order) then
$$S=sum_{k=1}^nfrac1{x_k^{-1}-1}=sum_{k=1}^nfrac{x_k}{1-x_k}
$$
and
$$2S=sum_{k=1}^nleft(frac1{x_k-1}+frac{x_k}{1-x_k}right)
=sum_{k=1}^n(-1)=-n.$$
$endgroup$
add a comment |
$begingroup$
$1$ and $x_1,ldots,x_n$ are the roots of $z^{n+1}-1=0$. They are
the $(n+1)$-th roots of unity.
Then $1/(z^{n+1}-1)$
has a partial fraction expansion of the form
$$frac1{z^{n+1}-1}=frac a{z-1}+sum_{k=1}^nfrac{b_k}{x_kz-1}.$$
Multiplying by $z-1$ and setting $z=1$ gives $a=1/(n+1)$.
Multiplying by $x_kz-1$ and setting $z=1/x_k$ gives $b_k=1/(n+1)$.
Therefore
$$sum_{k=1}^nfrac1{x_kz-1}=frac{n+1}{z^{n+1}-1}-frac1{z-1}.$$
Letting $zto1$ gives
$$sum_{k=1}^nfrac1{x_k-1}=lim_{zto1}frac{n-z-z^2-cdots-z^n}{z^{n+1}-1}
=frac{-1-2-cdots-n}{n+1}=-frac n2.$$
ADDED IN EDIT.
Here's a trick proof. Let $S$ denote the sum in question. As the
reciprocals of the $x_k$ are the $x_k$ again (in a different order) then
$$S=sum_{k=1}^nfrac1{x_k^{-1}-1}=sum_{k=1}^nfrac{x_k}{1-x_k}
$$
and
$$2S=sum_{k=1}^nleft(frac1{x_k-1}+frac{x_k}{1-x_k}right)
=sum_{k=1}^n(-1)=-n.$$
$endgroup$
add a comment |
$begingroup$
$1$ and $x_1,ldots,x_n$ are the roots of $z^{n+1}-1=0$. They are
the $(n+1)$-th roots of unity.
Then $1/(z^{n+1}-1)$
has a partial fraction expansion of the form
$$frac1{z^{n+1}-1}=frac a{z-1}+sum_{k=1}^nfrac{b_k}{x_kz-1}.$$
Multiplying by $z-1$ and setting $z=1$ gives $a=1/(n+1)$.
Multiplying by $x_kz-1$ and setting $z=1/x_k$ gives $b_k=1/(n+1)$.
Therefore
$$sum_{k=1}^nfrac1{x_kz-1}=frac{n+1}{z^{n+1}-1}-frac1{z-1}.$$
Letting $zto1$ gives
$$sum_{k=1}^nfrac1{x_k-1}=lim_{zto1}frac{n-z-z^2-cdots-z^n}{z^{n+1}-1}
=frac{-1-2-cdots-n}{n+1}=-frac n2.$$
ADDED IN EDIT.
Here's a trick proof. Let $S$ denote the sum in question. As the
reciprocals of the $x_k$ are the $x_k$ again (in a different order) then
$$S=sum_{k=1}^nfrac1{x_k^{-1}-1}=sum_{k=1}^nfrac{x_k}{1-x_k}
$$
and
$$2S=sum_{k=1}^nleft(frac1{x_k-1}+frac{x_k}{1-x_k}right)
=sum_{k=1}^n(-1)=-n.$$
$endgroup$
$1$ and $x_1,ldots,x_n$ are the roots of $z^{n+1}-1=0$. They are
the $(n+1)$-th roots of unity.
Then $1/(z^{n+1}-1)$
has a partial fraction expansion of the form
$$frac1{z^{n+1}-1}=frac a{z-1}+sum_{k=1}^nfrac{b_k}{x_kz-1}.$$
Multiplying by $z-1$ and setting $z=1$ gives $a=1/(n+1)$.
Multiplying by $x_kz-1$ and setting $z=1/x_k$ gives $b_k=1/(n+1)$.
Therefore
$$sum_{k=1}^nfrac1{x_kz-1}=frac{n+1}{z^{n+1}-1}-frac1{z-1}.$$
Letting $zto1$ gives
$$sum_{k=1}^nfrac1{x_k-1}=lim_{zto1}frac{n-z-z^2-cdots-z^n}{z^{n+1}-1}
=frac{-1-2-cdots-n}{n+1}=-frac n2.$$
ADDED IN EDIT.
Here's a trick proof. Let $S$ denote the sum in question. As the
reciprocals of the $x_k$ are the $x_k$ again (in a different order) then
$$S=sum_{k=1}^nfrac1{x_k^{-1}-1}=sum_{k=1}^nfrac{x_k}{1-x_k}
$$
and
$$2S=sum_{k=1}^nleft(frac1{x_k-1}+frac{x_k}{1-x_k}right)
=sum_{k=1}^n(-1)=-n.$$
edited Dec 13 '18 at 20:55
answered Dec 13 '18 at 19:37
Lord Shark the UnknownLord Shark the Unknown
105k1160133
105k1160133
add a comment |
add a comment |
$begingroup$
Applying the same technique as per this answer
$$Q(x)=prodlimits_{k=1}^n(x-x_k) text{ and }
Q'(x)=sumlimits_{i=1}^{n}prodlimits_{k=1,kne i}^{n}left(x-x_kright)$$
then
$$frac{Q'(x)}{Q(x)}=sumlimits_{i=1}^{n}frac{1}{x-x_i}$$
or
$$-frac{Q'(1)}{Q(1)}=sumlimits_{i=1}^{n}frac{1}{x_i-1}$$
But $Q(1)=n+1$ and $Q'(x)=1+2x+3x^3+...+nx^{n-1}Rightarrow Q'(1)=frac{n(n+1)}{2}$ and
$$sumlimits_{i=1}^{n}frac{1}{x_i-1}=-frac{n}{2}$$
$endgroup$
add a comment |
$begingroup$
Applying the same technique as per this answer
$$Q(x)=prodlimits_{k=1}^n(x-x_k) text{ and }
Q'(x)=sumlimits_{i=1}^{n}prodlimits_{k=1,kne i}^{n}left(x-x_kright)$$
then
$$frac{Q'(x)}{Q(x)}=sumlimits_{i=1}^{n}frac{1}{x-x_i}$$
or
$$-frac{Q'(1)}{Q(1)}=sumlimits_{i=1}^{n}frac{1}{x_i-1}$$
But $Q(1)=n+1$ and $Q'(x)=1+2x+3x^3+...+nx^{n-1}Rightarrow Q'(1)=frac{n(n+1)}{2}$ and
$$sumlimits_{i=1}^{n}frac{1}{x_i-1}=-frac{n}{2}$$
$endgroup$
add a comment |
$begingroup$
Applying the same technique as per this answer
$$Q(x)=prodlimits_{k=1}^n(x-x_k) text{ and }
Q'(x)=sumlimits_{i=1}^{n}prodlimits_{k=1,kne i}^{n}left(x-x_kright)$$
then
$$frac{Q'(x)}{Q(x)}=sumlimits_{i=1}^{n}frac{1}{x-x_i}$$
or
$$-frac{Q'(1)}{Q(1)}=sumlimits_{i=1}^{n}frac{1}{x_i-1}$$
But $Q(1)=n+1$ and $Q'(x)=1+2x+3x^3+...+nx^{n-1}Rightarrow Q'(1)=frac{n(n+1)}{2}$ and
$$sumlimits_{i=1}^{n}frac{1}{x_i-1}=-frac{n}{2}$$
$endgroup$
Applying the same technique as per this answer
$$Q(x)=prodlimits_{k=1}^n(x-x_k) text{ and }
Q'(x)=sumlimits_{i=1}^{n}prodlimits_{k=1,kne i}^{n}left(x-x_kright)$$
then
$$frac{Q'(x)}{Q(x)}=sumlimits_{i=1}^{n}frac{1}{x-x_i}$$
or
$$-frac{Q'(1)}{Q(1)}=sumlimits_{i=1}^{n}frac{1}{x_i-1}$$
But $Q(1)=n+1$ and $Q'(x)=1+2x+3x^3+...+nx^{n-1}Rightarrow Q'(1)=frac{n(n+1)}{2}$ and
$$sumlimits_{i=1}^{n}frac{1}{x_i-1}=-frac{n}{2}$$
answered Dec 13 '18 at 21:08
rtybasertybase
11.1k21533
11.1k21533
add a comment |
add a comment |
$begingroup$
We have $$x^{n+1}-1=0 (1)$$ with $xne1$
Set $dfrac1{x-1}=yiff x=dfrac{y+1}y$
Replace the value of $x$ in terms of $y$ in $(1)$ to form an $n$ degree equation in $y$ $$left(dfrac{y+1}yright)^{n+1}=1$$
$$impliesbinom{n+1}1 y^n+binom{n+1}2 y^{n-1}+cdots+1=0$$
Now apply Vieta's formula to find $$sum_{r=1}^ndfrac1{x_r-1}=sum_{r=1}^ny_r=-dfrac{binom{n+1}2}{binom{n+1}1}=?$$
$endgroup$
add a comment |
$begingroup$
We have $$x^{n+1}-1=0 (1)$$ with $xne1$
Set $dfrac1{x-1}=yiff x=dfrac{y+1}y$
Replace the value of $x$ in terms of $y$ in $(1)$ to form an $n$ degree equation in $y$ $$left(dfrac{y+1}yright)^{n+1}=1$$
$$impliesbinom{n+1}1 y^n+binom{n+1}2 y^{n-1}+cdots+1=0$$
Now apply Vieta's formula to find $$sum_{r=1}^ndfrac1{x_r-1}=sum_{r=1}^ny_r=-dfrac{binom{n+1}2}{binom{n+1}1}=?$$
$endgroup$
add a comment |
$begingroup$
We have $$x^{n+1}-1=0 (1)$$ with $xne1$
Set $dfrac1{x-1}=yiff x=dfrac{y+1}y$
Replace the value of $x$ in terms of $y$ in $(1)$ to form an $n$ degree equation in $y$ $$left(dfrac{y+1}yright)^{n+1}=1$$
$$impliesbinom{n+1}1 y^n+binom{n+1}2 y^{n-1}+cdots+1=0$$
Now apply Vieta's formula to find $$sum_{r=1}^ndfrac1{x_r-1}=sum_{r=1}^ny_r=-dfrac{binom{n+1}2}{binom{n+1}1}=?$$
$endgroup$
We have $$x^{n+1}-1=0 (1)$$ with $xne1$
Set $dfrac1{x-1}=yiff x=dfrac{y+1}y$
Replace the value of $x$ in terms of $y$ in $(1)$ to form an $n$ degree equation in $y$ $$left(dfrac{y+1}yright)^{n+1}=1$$
$$impliesbinom{n+1}1 y^n+binom{n+1}2 y^{n-1}+cdots+1=0$$
Now apply Vieta's formula to find $$sum_{r=1}^ndfrac1{x_r-1}=sum_{r=1}^ny_r=-dfrac{binom{n+1}2}{binom{n+1}1}=?$$
edited Dec 14 '18 at 4:41
answered Dec 13 '18 at 19:29
lab bhattacharjeelab bhattacharjee
226k15157275
226k15157275
add a comment |
add a comment |
$begingroup$
Simply note that $1,x_1,ldots, x_n$ are roots of $$
x^{n+1} -1 = 0.$$ Thus we have $0,x_1-1,ldots, x_n-1$ are roots of
$$
(x+1)^{n+1} -1 = xsum_{j=0}^{n} binom{n+1}{j+1}x^j = 0.
$$
Hence, $z_i = x_i -1$, $1leq i leq n$ are roots of
$$
sum_{j=0}^{n} binom{n+1}{j+1}z^j = 0,
$$ and by Vieta's formula,
$$
sum_{i=1}^n frac{1}{z_i} = frac{sum_{i=1}^n z_1z_2cdots z_{i-1}widehat{z_i}z_{i+1}cdots z_n}{z_1z_2cdots z_{n-1}z_n} = -frac{binom{n+1}{2}}{binom{n+1}{1}} = -frac{n}{2},
$$ where $widehat{z_i}$ means the variable is omitted in the calculation.
$endgroup$
add a comment |
$begingroup$
Simply note that $1,x_1,ldots, x_n$ are roots of $$
x^{n+1} -1 = 0.$$ Thus we have $0,x_1-1,ldots, x_n-1$ are roots of
$$
(x+1)^{n+1} -1 = xsum_{j=0}^{n} binom{n+1}{j+1}x^j = 0.
$$
Hence, $z_i = x_i -1$, $1leq i leq n$ are roots of
$$
sum_{j=0}^{n} binom{n+1}{j+1}z^j = 0,
$$ and by Vieta's formula,
$$
sum_{i=1}^n frac{1}{z_i} = frac{sum_{i=1}^n z_1z_2cdots z_{i-1}widehat{z_i}z_{i+1}cdots z_n}{z_1z_2cdots z_{n-1}z_n} = -frac{binom{n+1}{2}}{binom{n+1}{1}} = -frac{n}{2},
$$ where $widehat{z_i}$ means the variable is omitted in the calculation.
$endgroup$
add a comment |
$begingroup$
Simply note that $1,x_1,ldots, x_n$ are roots of $$
x^{n+1} -1 = 0.$$ Thus we have $0,x_1-1,ldots, x_n-1$ are roots of
$$
(x+1)^{n+1} -1 = xsum_{j=0}^{n} binom{n+1}{j+1}x^j = 0.
$$
Hence, $z_i = x_i -1$, $1leq i leq n$ are roots of
$$
sum_{j=0}^{n} binom{n+1}{j+1}z^j = 0,
$$ and by Vieta's formula,
$$
sum_{i=1}^n frac{1}{z_i} = frac{sum_{i=1}^n z_1z_2cdots z_{i-1}widehat{z_i}z_{i+1}cdots z_n}{z_1z_2cdots z_{n-1}z_n} = -frac{binom{n+1}{2}}{binom{n+1}{1}} = -frac{n}{2},
$$ where $widehat{z_i}$ means the variable is omitted in the calculation.
$endgroup$
Simply note that $1,x_1,ldots, x_n$ are roots of $$
x^{n+1} -1 = 0.$$ Thus we have $0,x_1-1,ldots, x_n-1$ are roots of
$$
(x+1)^{n+1} -1 = xsum_{j=0}^{n} binom{n+1}{j+1}x^j = 0.
$$
Hence, $z_i = x_i -1$, $1leq i leq n$ are roots of
$$
sum_{j=0}^{n} binom{n+1}{j+1}z^j = 0,
$$ and by Vieta's formula,
$$
sum_{i=1}^n frac{1}{z_i} = frac{sum_{i=1}^n z_1z_2cdots z_{i-1}widehat{z_i}z_{i+1}cdots z_n}{z_1z_2cdots z_{n-1}z_n} = -frac{binom{n+1}{2}}{binom{n+1}{1}} = -frac{n}{2},
$$ where $widehat{z_i}$ means the variable is omitted in the calculation.
edited Dec 14 '18 at 6:13
answered Dec 14 '18 at 5:52
SongSong
15.9k1739
15.9k1739
add a comment |
add a comment |
$begingroup$
First, the denominator is, I think, $(-1)^nQ(1)$.
Here is a hint for the numerator: how can you write $Q’(x)$?
$endgroup$
add a comment |
$begingroup$
First, the denominator is, I think, $(-1)^nQ(1)$.
Here is a hint for the numerator: how can you write $Q’(x)$?
$endgroup$
add a comment |
$begingroup$
First, the denominator is, I think, $(-1)^nQ(1)$.
Here is a hint for the numerator: how can you write $Q’(x)$?
$endgroup$
First, the denominator is, I think, $(-1)^nQ(1)$.
Here is a hint for the numerator: how can you write $Q’(x)$?
answered Dec 13 '18 at 19:31
MindlackMindlack
4,780210
4,780210
add a comment |
add a comment |
Thanks for contributing an answer to Mathematics Stack Exchange!
- Please be sure to answer the question. Provide details and share your research!
But avoid …
- Asking for help, clarification, or responding to other answers.
- Making statements based on opinion; back them up with references or personal experience.
Use MathJax to format equations. MathJax reference.
To learn more, see our tips on writing great answers.
Sign up or log in
StackExchange.ready(function () {
StackExchange.helpers.onClickDraftSave('#login-link');
});
Sign up using Google
Sign up using Facebook
Sign up using Email and Password
Post as a guest
Required, but never shown
StackExchange.ready(
function () {
StackExchange.openid.initPostLogin('.new-post-login', 'https%3a%2f%2fmath.stackexchange.com%2fquestions%2f3038472%2fif-x-1-x-2-ldots-x-n-are-the-roots-for-1xx2-ldotsxn-0-find-the-value%23new-answer', 'question_page');
}
);
Post as a guest
Required, but never shown
Sign up or log in
StackExchange.ready(function () {
StackExchange.helpers.onClickDraftSave('#login-link');
});
Sign up using Google
Sign up using Facebook
Sign up using Email and Password
Post as a guest
Required, but never shown
Sign up or log in
StackExchange.ready(function () {
StackExchange.helpers.onClickDraftSave('#login-link');
});
Sign up using Google
Sign up using Facebook
Sign up using Email and Password
Post as a guest
Required, but never shown
Sign up or log in
StackExchange.ready(function () {
StackExchange.helpers.onClickDraftSave('#login-link');
});
Sign up using Google
Sign up using Facebook
Sign up using Email and Password
Sign up using Google
Sign up using Facebook
Sign up using Email and Password
Post as a guest
Required, but never shown
Required, but never shown
Required, but never shown
Required, but never shown
Required, but never shown
Required, but never shown
Required, but never shown
Required, but never shown
Required, but never shown
$begingroup$
Related : math.stackexchange.com/questions/2988339/…
$endgroup$
– Arnaud D.
Dec 17 '18 at 12:07